Instigator / Con
4
1644
rating
64
debates
65.63%
won
Topic
#2637

Resolved: Violent revolution is a just response to political oppression

Status
Finished

The debate is finished. The distribution of the voting points and the winner are presented below.

Winner & statistics
Better arguments
0
3
Better sources
2
2
Better legibility
1
1
Better conduct
1
1

After 1 vote and with 3 points ahead, the winner is...

MisterChris
Parameters
Publication date
Last updated date
Type
Standard
Number of rounds
4
Time for argument
One week
Max argument characters
15,000
Voting period
Two weeks
Point system
Multiple criterions
Voting system
Open
Contender / Pro
7
1762
rating
45
debates
88.89%
won
Description

Revolution: As a historical process, “revolution” refers to a movement, often violent, to overthrow an old regime and effect, complete change in the fundamental institutions of society -- http://www.columbia.edu/cu/weai/exeas/asian-revolutions/pdf/what-is-revolution.pdf

Violent: using force to hurt or attack, used to describe a situation or event in which people are hurt or killed -- https://dictionary.cambridge.org/us/dictionary/english/violent

Political: Relating to the activities of the government, members of law-making organizations, or people who try to influence the way a country is governed (https://dictionary.cambridge.org/us/dictionary/english/politics)

Oppression: a situation in which people are governed in an unfair and cruel way and prevented from having opportunities and freedom

Just: morally correct (similar to Justice: righteousness, equitableness, or moral rightness)

Round 1
Con
#1
Thanks, MisterChris. My argument will be from four perspectives: morality, practicality, effectiveness, and philosophy, to prove violent revolution to be unjust overall.

I. Contradiction of Violence 

For citizens to have to resort to violence is too much…
      Human life is invaluable, and one of our essential rights. But while it is arguable whether humans can fight for their freedom with their lives at stake, the inherent paradox of violating the rulers’ lives occurs as a result. Political oppression is often hidden behind red tape, a massive bureaucracy that is most likely nonviolent in nature. To strike back merely due to intangible laws and no physical harm induced becomes a questionable result. After all, even the official law only allows self-defense with evidence that the other party had shown intentions of harming. There must be an imminent threat, and reasonable force of harm, both of which are lacking in the majority of political oppression. [3]
        The additional problem is that while the revolution itself is the response to political oppression, the resulting actions can inhibit other people’s rights and desires. A journal on philosophy and public affairs states, “revolutionaries frequently use violence… against … other oppressed people… to increase participation in the struggle or to eliminate rivals”. [5, p293]
       Once you have felt so angered that you feel you must physically act, I see no inherent barrier preventing you from doing the same to others who fail to act. The nature of violence sets the other as an enemy; you would only inflict pain on those who have seriously violated your rights. You are far more likely to see the world as a dichotomy of two views set against each other and believe that anyone NOT on your side, is on the enemy’s side. The enforcement of your belief through violence re-enforces the idea that you must not tolerate even the slightest hesitation. 
          The ending point I’m making is this: the inherent nature of the resulting conflict is violent in itself. If the people begin it, they would arguably be further at fault, justifying a bloody result and preventing other humanitarian countries from intervening. You would be less likely to turn over someone from the opposing side who may or may not be on the fence, as you would have to resort to torture and force them by coercion, rather than persuading them with kindness and need for sympathy. 
       This point may seem short, but the fundamental morality of using violence as a justification for ambiguous standards of “unfair oppression” means the people would be able to judge however they want and punish with severe physical harm or even death. The checks and balances widely exist among democracies to prevent this from occurring, and merely nonviolent protest is enough for the government to change their ideals. The authorities understand that worker strikes and taking their time up will waste government resources and bring a slight dent to the economy. The people’s voice matters inherently and will change election results. The feedback system works very well, and Pro must tell us why it is not enough.

II. War vs Revolution

        This point might look self-defeating. For clarification, I am saying that war may be ironically more effective than people’s violent revolution from within. The logic is that, if we are forced to create a conflict, it is far more effective for official war and cut off public support for authorities of the country. If the US declared Hitler public enemy number one, this is more effective (ends it faster), more beneficial (saves more lives), and therefore just, unlike citizens wasting their lives away, untrained battling the trained regiment. The citizens ought to merely wait peacefully to avoid sacrificing their lives needlessly. MisterChris can only win this if he proves violent revolution is the only way out.
          Knowing MisterChris, he will likely come upon the multiple examples of ways we strived to achieve equality in times of difficulty by using violence to achieve our results. Perhaps the most famous is the American civil war. By Cambridge’s definition, the illusion seems to be that the North and the South fought each other to overcome the inherent regime and establish their own beliefs. But delving deeper, the confusion of civil war versus revolution becomes apparent. 
        Aristotle from times of Greek had “defined revolution as a fundamental change in the state organization or the political power, which takes place in a short time and that entails a revolt of the population against the authority” [1]. 
        However, the south had officially and legally succeeded from the union during the civil war. So the difference between a usual war is blurred, and MisterChris could not use these specific examples as people fighting for their rights. When the people fight the people, there is a failed connection of defeating the actual authority. Certainly, the president led the North in the time of the trouble, but there was arguably just as many people in the south, not to mention, the authority won in this case, thus negating any potential ideas that inherently suppose the violent “revolution” would be just, from point of view of the south.

       For MisterChris to succeed, he must not only prove the worth of benefits gained from Revolutions (and superior to other methods), he would have to justify it differing from other forms of war that are inherently different. My same source notes the differences for the causes: 
      “The elements [of revolution] include opposition among the elites, feeling of resistance within the masses, suitable international relations, widespread anger within the population, and economic or financial imbalances. Conversely, civil wars appear to be triggered by greed (i.e. individuals seek to maximize their profits), grievance (i.e. there is a social and political unstable equilibrium), and opportunities (i.e. social inequalities, poverty, oppression, etc.);”
           Linking this back to my point: the worry I have is that the people themselves shouldn’t need to respond. It would be unjust to have the citizens fight against trained soldiers if others could help. So MisterChris also has to give a convincing case why we can’t ask for help from allies, neighboring countries, or even the UN. After all, the “police of freedom” US is already assisting and changing the idea of mere violent revolution, into a full on war, to enforce fully the idea that the authority is cruel and ought to be condemned. 
            Here’s a useful comparison: If a citizen had to perform open-heart surgery with only a 10% chance of success and nobody could help him, yes, it is just for him to decide to perform open-heart surgery. But if a foreign doctor was here and could perform open-heart surgery with 80% of success, surely, the foreign doctor should go here, help him out. So it would be unjust to let the citizens perform open-heart surgery here.

III. Failures of Violent Revolution
     Nonviolence is well renowned to trump violence in the vast majority of cases. Chenoweth is a famous researcher who analyzed 323 different violent and nonviolent revolutions since 1900. The results? Nonviolent campaigns have a 53% success rate and only about a 20% rate of complete failure, while violent campaigns were only successful 23% of the time, and complete failures about 60% of the time. [2] Even looking only at partial successes, nonviolent campaigns still won out in the end (20% vs 10%). Now you might say, okay, there’s a correlation, where’s causation? 
        No worries, my expert here agrees with me. The logical reasoning is that it “enhances its domestic and international legitimacy and encourages more broad-based participation...recognition … can translate into greater internal and external support for that group… undermining the regime’s main sources of political, economic, and even military power.” [4, p9] Now, at last, you see the core of my argument. If you agree with Arg 1, then you can see how the unnecessary amount of deaths are ridiculous. Other countries are crucial nowadays for survival, whether it be through trade, through immigrants, among countless other factors. Even with disagreeing against Arg 2, you can still agree that the countries may take non-violent responses to slowly whittle and destroy the country without a full-on war. 
         In Chenoweth’s own words, the forcible unjust acts of political oppression may become obvious if they are forced into the realm of violence first. While violent revolution often requires us to act first, the subtle nature of nonviolent revolution allows for greater power shifts. Unlike being able to label the violent revolution response as terrorism, the nonviolent nature of the people are harder to destroy inherently, and therefore just. The empirical evidence combines with logic reasoning and upholds the burden.

Now onto Con...tradiction of revolution. [IV]

     Small pun aside, you might be wondering how this differs from the point I. The logic here is that in the cases where the causes may be justified, the result cannot be accomplished (and vice versa). So firstly, we already know from II that it may be a bad idea for citizens to battle-trained soldiers. It requires unconventional attacks to truly take down the regime, for example, administration or non-military based agency. However, by the nature of terrorism, you would be pitted as a general enemy of the state and the people. It’s highly unlikely that anyone would join you on this front. Surely there’s a better way to go around this. 

Indeed, it’s possible but more difficult, to succeed without using terrorism. But achieving the leadership for the small rebellion and coordinating the violence is painstaking. Why? The regime would already dissatisfy the requirements of freedom of assembly, expression, etc. The task to gain a trusted leader in secret would be an insurmountable goal. The Syria revolution, for example, failed to gain international support, along with charismatic leadership necessary to rally the people together. [6] So the violent revolution falls apart at its core, unable to cause change for the better. 

Connecting back to I, the active participation by a significant amount of people is problematic. For instance, securing the actual goods and weapons necessary to launch the attack would be difficult. Most people are motivated by self-interest, and therefore, they would be stuck at an impasse where each person knows as long as someone else takes their spot, the revolution may succeed, yet ironically, some person needs to self-sacrifice for the collective. Each person’s participation may be limited based on how much gain they expect to gain. And each individual’s rationality would prevent them from participating in the revolution, especially considering cost and benefit. Not to mention the regime can always raise the cost higher at any time, making it difficult for a truly successful and motivated revolution. 

The use of emotion as a powerful weapon is inherently dangerous, even in cases where violence is not used on non-combatants. By nature of the difficult sacrifice of violent revolution, the moral agent must have strong enough motivation. This is a self-evident truth that needs no backing. As a result, violent revolutions often stem from misinformation and slander of the opposing force. 

The American Revolution is perhaps the most famous one. Though the revolution itself may be justified, the causes and the lack of fully informed people make it dubious if the methods used to achieve it were correct. A book exposes how Ben Franklin wrote a fake leader on how the Britain king “engages savages to murder their defenseless farmers, women, and children.” [7] Now tell me, is it just, to rally your people with false cause and create political oppression? Nowhere in the premise does it say, the political oppression exists. Pro may argue, “wait a minute, wouldn’t other people notice”? Well. We didn’t even notice the false information about the American Revolution until this book’s release when it’s far too late to condemn Ben Franklin. So Pro must now overcome the potential fallibility of people allowed to determine and judge the standard for “oppression”, against all facets of false news and information.

Finally, Pro will probably say, yes, all of these and more, but emergencies necessitate the people HAVE to act now as any difference is better than none. Well, I say that nonviolence is inherently not punishable by a rational regime; the annihilation or slavery of citizens would not occur to innocent people, and a sufficiently powerful regime would negate any effect of small violence on the people’s part, reducing the seemingly powerful act for liberty to meaninglessness. You could argue that the very fact that no amount of violence could truly reduce the power of the regime, gives people a lack of hope. By keeping quiet and biding your time, you demonstrate your will to fight on in your way.

Conclusion: Gandhi was right: “an eye for an eye makes the whole world blind”. Either violent revolutions are just, despite their tainted history, their causation of bloodshed, their ineffectiveness, their illogical nature, and coercion against people, or revolutions are unjust overall.

  1. http://www.differencebetween.net/miscellaneous/difference-between-civil-war-and-revolution/ 
  2. https://www.psychologytoday.com/us/blog/sex-murder-and-the-meaning-life/201404/violent-versus-nonviolent-revolutions-which-way-wins 
  3. https://criminal.findlaw.com/criminal-law-basics/self-defense-overview.html#:~:text=Self%2Ddefense%20law%20requires%20the,force%20to%20counteract%20the%20threat
  4. https://www.jstor.org/stable/40207100?seq=3#metadata_info_tab_contents 
  5. https://www.jstor.org/stable/42703863?seq=3#metadata_info_tab_contents 
  6. https://www.middleeasteye.net/opinion/who-really-blame-failure-syrian-revolution
  7. https://www.newswise.com/articles/founding-fathers-used-fake-news-racial-fear-mongering-to-unite-colonies-during-american-revolution-new-book-reveals


Pro
#2
Thx, Undefeatable.

Resolved: Violent revolution is a just response to political oppression.

OBSERVATIONS:

  • The debate structure as agreed upon can be found here
  • CON defines “just” as morally correct. Well, in this scenario, what does a “morally correct” response entail? A more detailed definition of “just” by Ethics Defined states: “Justice is considered post fact; as a means of reciprocity – correcting the scales to provide some equilibrium of fairness.”  “Political oppression” in the resolution makes the agent of oppression the government, tipping the scales out of balance. A “just,” “morally correct” response therefore would be anything that attempts to correct those scales to a proportional degree. This does not entail that the response would have to be successful to be “just,” hence the word “attempts,” but instead it requires us to consider whether the attempted response is proportional to the injustice it attempted to correct.
  • Along this line, to be a proportional response in this scenario, violent revolution must be done against a government where it has been deemed necessary by an exasperated populace that feels as if they have no other choice. There are an arsenal of options for people to take to combat political oppression, such as trying for reform and civil disobedience. Declaring war against the state is a heavy matter that puts the lives of the revolutionaries and their families on the line, people do not undertake such a risk without having exasperated all other options. Indeed, as soon as those peaceful methods inevitably fail in the most tyrannical of governments, or they appear to have no utility against them, violent revolution then becomes the last-resort in the arsenal of options of the oppressed. Thus, CON must prove that violent revolution would be unjust even in the scenario where there seems to be no other choice for the oppressed (i.e. CON must prove in all scenarios, violent revolution is unjustifiable), while PRO must prove that violent revolution should be a tool in the arsenal of the oppressed in case they deem its use necessary.
CONTENTION 1: SELF-DEFENSE

There is no scenario where a populace rises against their oppressors violently, and it was not a form of self-defense. Indeed, it is self-defense, whether the oppressed is rising up against physical or structural violence, or both.

Now, it’s easy to see how the former is self-defense. In an op-ed by Stanford Daily, they write:
“If someone is gunning down your entire family, few would condemn you for fighting back in order to protect your remaining loved ones. So if the state is killing your community – your extended family, everyone who looks like you – why should anyone condemn you for trying to protect them?”

As for the latter, it is more difficult.
Structural violence is defined as “a form of violence wherein social structures or social institutions harm people by preventing them from meeting their basic needs.”

Wikipedia gives a good synopsis on the subject:
“In his book Violence: Reflections on a National Epidemic, James Gilligan defines structural violence as "the increased rates of death and disability suffered by those who occupy the bottom rungs of society, as contrasted with the relatively lower death rates experienced by those who are above them". Gilligan largely describes these "excess deaths" as "non-natural" and attributes them to the stress, shame, discrimination, and denigration that results from lower status…”

Thus, Stanford furthers:
“Why does structural violence not merit the same urgency (as physical violence)?
If anything, the urgency is only greater. As opposed to a single punch or gunshot, structural violence is unwavering...You can disagree with the strategic efficacy of their actions, but when people defy — even violently — this endless structural violence, the only consistent position is to label their defiance self-defense.”

Indeed, as Paulo Friere writes in Pedagogy of the Oppressed,
“With the establishment of a relationship of oppression, violence has already begun. Never in history has violence been initiated by the oppressed. How could they be the initiators, if they themselves are the result of violence? How could they be the sponsors of something whose objective inauguration called forth their existence as oppressed? There would be no oppressed had there been no prior situation of violence to establish their subjugation.
Violence is initiated by those who oppress, who exploit, who fail to recognize others as persons—not by those who are oppressed, exploited, and unrecognized”

RECALL that “declaring war against the state is a heavy matter that puts the lives of the revolutionaries and their families on the line, people do not undertake such a risk without having exasperated all other options.”

We can assume that if people undertake a violent revolution because of structural violence, it was because the endless cycles of oppression reached a boiling point. 

CONTENTION 2: PACIFISM VS. TRUE TYRANNY

While nonviolent methods of change can be effective, that privilege is limited to the majority of societies of the past few centuries where, despite some injustice, people are otherwise generally free to do as they please. But in a country that has true, concerted repression... well, it's a different story.

As Howard Ryan writes: “The difficulty of maintaining a nonviolent struggle in face of concerted repression is too often minimized by proponents of nonviolence. The failure of nonviolent campaigns has been attributed to the "lack of discipline" of protesters and their inability to withstand suffering, rather than to inherent problems involved in a movement that does not defend against repression.” 

In order to understand this further, one must understand what kind of situations violent revolution would be necessary. 

First Communist China, where the citizens have been ruled with an iron fist even now:
"At home, the Chinese Communist Party, worried that permitting political freedom would jeopardize its grasp on power, has constructed an Orwellian high-tech surveillance state and a sophisticated internet censorship system to monitor and suppress public criticism. Abroad, it uses its growing economic clout to silence critics and to carry out the most intense attack on the global system for enforcing human rights since that system began to emerge in the mid-20th century...

No other government is simultaneously detaining a million members of an ethnic minority for forced indoctrination and attacking anyone who dares to challenge its repression. And while other governments commit serious human rights violations, no other government flexes its political muscles with such vigor and determination to undermine the international human rights standards and institutions that could hold it to account."

Nonviolent attempts in the past to change the system in China have been thwarted violently.
For example, during the 1989 Tiananmen Square protests, the State Council declared martial law, and mobilized as many as 250,000 troops to Beijing. The troops ruthlessly suppressed the protests by firing at demonstrators with automatic weapons, killing hundreds of protesters and leading to mass civil unrest in the days following.

Second, it is difficult to imagine nonviolence being viable for Jews against the socialist government in Nazi Germany, where according to Pacifism as Pathology by Ward Churchill:
“it has been abundantly documented that the Nazi policy toward the Jews, from 1941 onward, was that extermination would proceed until the entire Jewish population within German territory was liquidated. There is no indication whatsoever that nonviolent intervention/mediation from any quarter held the least prospect of halting, or even delaying, the genocidal process. To the contrary, there is evidence that efforts by neutral parties such as the Red Cross had the effect of speeding up the slaughter. That the Final Solution was halted at a point short of its full realization was due solely to the massive application of armed force against Germany.. Left to a pacifist prescription, Jews would have suffered total extermination by mid-1946 at the latest.”

More recently, Howard Ryan writes:
“In El Salvador today, where public protests are broken up with guns and riot gear, worker and peasant organizers are arrested and tortured, and villages are brutally massacred. To expect Salvadorans, under such intense attack, to continue their struggle for justice nonviolently seems neither realistic nor humane.”

Need I even mention North Korea?

The list simply goes on of instances in which the only viable means to liberty is self-defense through violent revolution, and to take that option away from the oppressed is to impede on their ability to achieve justice. 

CONCLUSION:

The oppressed reserve the right to revolt violently against a government where there seems to be no alternative… Indeed, not only is it self-defense, but it is perhaps the only way out of their oppression.

To quote PRO’s 3rd observation:
“there are an arsenal of options for people to take to combat political oppression, such as trying for reform and civil disobedience. Declaring war against the state is a heavy matter that puts the lives of the revolutionaries and their families on the line, people do not undertake such a risk without having exasperated all other options. Indeed, as soon as those peaceful methods inevitably fail in the most tyrannical of governments, or they appear to have no utility against them, violent revolution then becomes the last-resort in the arsenal of options of the oppressed.”





Round 2
Con
#3
VR = Violent Revolution

CHALLENGE FRAMEWORK

Pro makes two excellent arguments that implicitly attack points I and III. Here I will respond by constructing the framework that Pro must successfully fulfill, or refute, to win this debate. Firstly, though the Burden of Proof is shared, I do not have to show it is unjust in all manner of cases. It would be too easy for Pro to list one single example where the regime was ruthless, yet stupid enough to fall for the sneak attacks of the revolutionaries. A perfect VR where sacrifices were minimal and everything went swimmingly. I will layout what requirements he must show that VR's have as a whole.

There are a few crucial requirements for VR to be just:
1) There must be sufficient reason (fulfilled by Pro, BUT problematic)
2) There must be a sufficient level of success (unfulfilled by Pro-- he only showed nonviolence relatively unsuccessful)
3) There must be no other option (point II is not completely batted away from Pro's ideas)
4) The benefits must outweigh the negatives (unfulfilled by Pro)
5) The revolutionaries must be able to consider moral agents (more on this later, unfulfilled by Pro)

Firstly, why is 1) Problematic? The Regime might be more subtle and difficult in its actual oppression. Outright violence does not have to be fulfilled for people to feel pressured. Look at Taxation without representation, a vague idea, compared to recently, the cop kneeling on George Floyd until he couldn't breathe. Indeed, the people are the judge and standard for the "political oppression". Already, we see my final point of IV coming back into play. Yes, you can point out the structural racism existing in the US. But is it truly the government choosing to be racist and enacting actual laws encouraging cops to kneel on George Floyd? There is certainly a reason, but the VR's justification seems ridiculous. Ransack the innocent stores, hit all the police -- racist or not-- with smoke bombs, yes, this will lead to structural racism stopping! No. The misinformation is problematic especially if exaggerated, and the "political oppression" is thus impossible to truly determine upon the standard unless it is astoundingly obvious (intruding on the majority of citizens' rights to life, liberty, and pursuit of happiness).

Pro may ask, why is 2) necessary for VR and not for self-defense? The reasoning is logical. With individuals' lives at risk, their success is not profound to their justification. They are indeed striving for change, but with abuse and oppression in their face as an alternative, the worst-case scenario is that they are damned if they do, damned if they don't. However, VR's are inherently different. They are bringing other people and convincing them to battle the regime. Here I bring attention to Pro's actual arguments. He points out that nonviolence is utterly useless against powerful enough regimes. But as I said myself, under these extreme situations you are merely martyrs sending yourself to your doom, if only bringing the message that you are desperate and violent. He must not only show that VR is more successful than non-VR, but also that VR's are successful in these special situations as well. Remember the example I gave about Syria-- the regime is so powerful that even the US fears it and refuses to help internationally. Neither VR's nor non-VR's help here. So VR's remain unjust.

Recall the numerous obstacles preventing VR in a powerful regime listed in R1:
  • Leadership and coordination
  • Being branded as a terrorist 
  • Access to weapons and Goods
  • Selfishness and reluctance of cost/benefit
  • Misinformation linked back to 1 that makes it hard to justify 
  • Battling trained soldiers while being untrained

With regards to 3), Pro points out that it is the final resort after VR, but misses out on my point regarding international relationships, rather choosing to focus on the civil war portion that I mentioned to make the point that War was different from VR. Unless he can defeat this idea where the "foreign doctor" should intervene, he fails to fulfill his burden of proof.

With regards to 4), note how Pro carefully chose to avoid listing VR's actual ability to change and the resulting society -- if it is better than before. There are numerous counter-studies to prove that VR's with their emotionally driven minds cannot result in a much better result. Even from the abstract of expert Mr. Weede's article notes, "In our view, revolutions do not contribute to the promotion of liberty, they merely generate a new ruling class or oligarchy, stronger armed forces, and more war-involvement. Whether evaluated by economic growth rates, income inequality, or quality of life, the economic performance of post-revolutionary regimes looks unconvincing. Certainly, it cannot compensate for the extraordinary loss of life resulting from 20th-century revolutions. " [1] As you can see, the revolutionary's ideals can also be problematic, and result in a never-ending cycle that Pro is so afraid of. VR leads to more VR's that ultimately destroy the country from within. 

Because this is the only expert source I could find on the topic, I will wring it dry to prove it trustworthy. Firstly, Weede points out that the VR's lead to negative economic impact (especially concerning growth, as distribution only solves relative and not absolute poverty), and constrains men and women both, causing the same old oppression if only new rule. Next, Weede lists countless examples where VR only led to more oppression, with Russia becoming worse; Castro's claim of power refusing migrants and refugees, and somehow, Khmer Rogue's rule was as bloody, if not more, than Hitler's and Stalin's. He finally concludes with Gurr's notice that revolutionaries tend to result in police states that are worse off than the original, listing more than a dozen examples.

The scholar adds more ideas to help enlighten point IV I made above (and a bit of point I), the most complex and convincing point I've made thus far. On page 331, he notes that the people are ready to kill. They must support the inherent ideas of ruthlessness and cruelty. From the rationality basis, you are able to sympathize much better with the non-cruel side than the cruel side. As such, the leading *impact* which I forgot in R1 is that people will now be just as ruthless as before. Remember my claim:

Once you have felt so angered that you feel you must physically act, I see no inherent barrier preventing you from doing the same to others who fail to act. The nature of violence sets the other as an enemy; you would only inflict pain on those who have seriously violated your rights. You are far more likely to see the world as a dichotomy of two views set against each other and believe that anyone NOT on your side, is on the enemy’s side. The enforcement of your belief through violence re-enforces the idea that you must not tolerate even the slightest hesitation. 
Unless Pro manages to prove that the revolutionaries have some obligation post-rule and won't be power hungry, this point stands upon its reasoning. 

Next, Weede notes the numerous problems with actually sustaining a well-made new government. The power hungry leaders are only there to keep power, and most times fail to think of actual constitutional laws and ideals to help the people. And once they become the actual leaders, they view themselves as those who deserve power -- a haunting similarity to the dictatorship. Despite the oligarchy in place, revolutionary regimes become exactly what they spoke up against. Knowing the incredible power of violence, the temptation is difficult to resist, and who's going to stop them? The "revolutionary revolutionaries"? 

Now, what the heck is 5)? Well, Pro assumes that everything is either just or unjust. I disagree. Note how he used the stronger word, "necessary". What is necessary? It cannot be "unjust" to merely breathe, have your heartbeat, be living, necessary ideals for your life. It is just to have the right to life. But is it considered "morally correct" to exercise the autonomous functions that let you survive? You are indeed exercising this just right. But the exercise of this right in itself is ambiguous and a naturally selected process. I assert that the basic functions that let you live and respond to a natural stimulus are neither just nor unjust-- they merely are. As pro asserts our inherent rights more and more, and he refutes my points, we must go back to the central question and ask as well -- can the person be considered a moral agent? After all, even other animals that do not have human rights are allowed to fight amongst each other. It is part of the natural state.

So here I go back to Locke and Hobbes' original ideals. They believe that people have the inherent right to say back against the government when something goes wrong in the contract. Of course, Pro could argue that I'm undermining my argument. The majority of people disagree with VR, and even still a great number of people would disagree with non-VR. The political and legal justification is different from the moralistic justification, however. I daresay that as Pro argues for the nature of self-defense, this oh-so-natural response could be considered neither just nor unjust. With these strange responses taken out of the impulse of human nature, the law should of course sanction these actions that are natural to men -- even if not morally supported.

The question of whether VR is ethically or morally reasonable may be irrelevant given its inevitability. Viciousness is a characteristic human drive that is escalated concerning political mistreatment. The state has an imposing business model on real brutality, yet this isn't simply equivalent to controlling it; rather, human violence is its power which the state can just control through a tricky plan. At the point when the state practices savagery for the sake of equity, it makes such a fiction by which the state's brutality is merely a response to the individuals' VR. At the point when the state's activity of viciousness seems to disregard the theoretical rules that legitimize it, at that point the state claims that protest and is presented to the chance of revenge.

This is because the main quality of brutality itself is its infectiousness. Savagery is practically never just or advocated. Here both the victim and oppression are pulled out of their societal aspects and onto the ontological level. Both the experience of torment and the experience of causing torment are basic encounters which make it hard to go back to rationality. There is a profoundly attached craving to attack the fire with fire (the evolutionary response I stated), a longing which exists preceding any legitimizations.

The response to violence could result in a whole new level. This is particularly evident given that the ensuing casualty comes from fluctuating levels of relationship with the first culprit, and the mischief incurred is regularly heightened. A man takes a goat; the proprietor of the goat strikes the man's child; the man's child murders the proprietor's significant other, etc. until all social qualifications have dissolved and no one can recall how the viciousness even began.

The state's imposing model on viciousness depends on theoretical guidelines accurately because a reflection gives the ideal of extreme reasoning for brutality which can be fulfilled in a solitary demonstration, without impelling a pattern of responded savagery without end. We can't comprehensively characterize something as conceptual as "equity" however we endeavor to diagram it completely enough for individuals to accept that the state's savagery for the sake of "equity" is likewise the fulfillment of their want for retributive viciousness. If we invest enough energy investigating the idea of "equity" at that point we rapidly find that the state's viciousness really can be interpreted as a reprisal against the individuals, and individuals would then discover the drive inside themselves to fight back with their brutality against the state.

My point here is that when you are truly talking about the chance of progressive savagery against the state in a way that isn't just theoretical, at that point no doubt some type of viciousness is now inescapable (instead of merely just or unjust). When the motivation is there, it needs someplace to go. No rationality will have the option to disperse it. Particularly if only the state opposes the ideals for VR.

Weighing of ideas

CONTENTION 1: SELF-DEFENSE
Pro says:
  • if the state is killing your community ... why should anyone condemn you for trying to protect them?
Outweighed by impossibility to succeed, impacts of revolution
  • Never in history has violence been initiated by the oppressed
Empty assertion and ignores the idea that oppression may be nonviolent (taxation without representation, heck, American Revolution. "Those confirmed to be on the scene to witness the first shots say the Americans fired first") [2]

CONTENTION 2: PACIFISM VS. TRUE TYRANNY
Pro says:
  • Nonviolent attempts in the past to change the system in China have been thwarted violently.
  • it is difficult to imagine nonviolence being viable for Jews against the socialist government in Nazi Germany
  • In El Salvador today, where public protests are broken up with guns and riot gear, worker and peasant organizers are arrested and tortured, and villages are brutally massacred.
All of these are irrelevant right now, as they are attacking non-VR while failing to support the idea that VR would actually work in these situations.

Conclusion: 

Pro has clearly done his study, but not on an overarching powerful level that fulfills all reasons necessary to initiate the VR, and also have it succeed. The problem is that he tries to support it from a moral perspective, but is trapped by my constructive to think the illusion that VR is either just or unjust. How do we actually know "political oppression" in more ambiguous cases? He also fails to think, can I actually start the VR? If you can't even begin the VR, then VR cannot be just. Next, he fails to note the actual impacts of VR -- does it work, does it not? And if it does overthrow the government, what happens next? Everyone lives happily ever after? Weede certainly doesn't think so. While usually my constructive is the most powerful part of my argument, I believe this refutation is stronger and enough to highlight how VR is not just. 



Pro
#4
Thx, Undefeatable.

R2 OBSERVATIONS:

  • RECALL that according to the debate structure, PRO is only permitted to respond to CON’s R1 this round. 
REFUTATIONS:

A/2 I. Contradiction of Violence 

1: Violence against non-physically violent regimes is immoral

  • CON argues that since political oppression is most likely to appear in a form that does not entail physical violence, we can no longer classify revolution as self-defense, and thus it is immoral. Notice that CON conveniently only addresses revolutions that use violence against non-physically violent regimes. This glaring hole is even acknowledged, but simply brushed off by CON: “it is arguable whether humans can fight for their freedom with their lives at stake.” 
While CON can pretend they don’t exist, the truth is, political oppression often takes physically violent forms. Under CON’s own self-defense framework, then, this instantaneously grants PRO the win. 

Now, the clear objection is: “I do not need to prove that violent revolution is immoral in every instance, only on an “on balance” basis. Therefore because most oppressive governments are not killing people openly, I win." This objection doesn't work for two reasons.

First, because such an “on balance” standard would introduce absurdity into the equation. Under the standard, those who were being slaughtered would be forced to simply roll over because of the potential prospect of other people in other places abusing the privilege of being able to revolt violently. 

Second, because the blanket standard of “if the response is more severe than the transgressions, it’s wrong” doesn’t work. In order for a populace to even be considering violent revolution, the transgressions against them must be severe and sustained, and they will have already tried nonviolent means to begin with. Is it not justified if someone has pinched you painfully every second of every day for the past year to give them a quick punch in the face to stop it? 

  • RECALL & EXTEND PRO’s 1st Contention. Violence as a whole isn't just direct, physical violence. Violence also includes structural violence, which by definition makes even the revolutions against non-physically violent yet oppressive regimes self-defensive in nature. 
CON may argue that indirect violence is not enough to justify direct violence by the populace. 

However, expanding on the “pinching point” earlier, if we use a Responsibility-Sensitive Account of Proportionality as posited by Zsolt Kapelner, we see that there are many scenarios where this is justified.  

He argues: “proportionality in case of defensive harm is sensitive to the degree of responsibility on the part of threatening agents. The more responsible one is, the less strict the proportionality requirement becomes… (i.e.) defence against agents who are responsible for posing a threat may involve greater harms than the harms with which the responsible agent threatens. For example, if one deliberately poses a threat of wrongful and painful pinching, call this person the Annoying Pincher, one might think that responding by inflicting greater harms than pinching, e.g. pushing or punching, is justified, although killing or torture are not. In contrast, someone who is addicted to pinching, call her the Addicted Pincher, and as such is much less responsible for posing a threat, may not be subjected to harms greater than pinching itself.”

Given this framework of proportionality, we can see that a populace that is being repeatedly targeted with structural violence would be justified in a more substantial response to cease the otherwise unobstructed pattern of indirect violence. 

2. Some innocents are harmed

CON makes a hypocritical argument here. They say violent revolution has a tendency to target innocents with violence, therefore it is immoral to undertake violent revolution at all. Yet, later on, CON argues that war between nations is a moral response to the oppression of a regime. The violent nature of violent revolution is not unique to violent revolution. Look at any war, even those that are commonly known to be justified, and you will see innocent parties caught in the crossfire of even the “good” parties. There were incidents of rape in France, for example, by US soldiers fighting the Nazis in WWII. Yet, no one will say that the US was unjustified to be fighting the Nazis. 

So clearly, then, CON’s analysis is off. The problem with his argument is that it is a moral absolutist view, in which any collateral damage by the means invalidates the ends, even if the ends outweigh the means in the long run. War is not fun for anyone involved, and there will be many individual injustices in any war. By this logic, no war is justified. Instead, we should adopt the view of situational morality, where the attempted ends justify the means. RECALL: “This does not entail that the response would have to be successful to be “just,” hence the word “attempts,” but instead it requires us to consider whether the attempted response is proportional to the injustice it attempted to correct.”

3. Violence prevents people from being sympathetic to the cause

Perhaps Howard Ryan says it best:

“If one's cause is clearly just, it isn't necessary to suffer peacefully the brutalities of the government in order to win popular support. To fight violently in defense of one's life, one's home, or one's freedom is a widely respected human right. In fact, where a movement faces heavy government repression, people may more likely join the struggle if resisters are fighting to defend their lives rather than allowing themselves to be beaten and punished. A campaign which emphasizes voluntary suffering may alienate rather than attract public support and participation. The question of which method will draw greater public support is an historical one: it cannot be answered in the abstract. Often, violent action would be a disaster; at other times, refusal to use violence may be disastrous. It depends on the situation.”

A/2 II. War vs Revolution

This is a self-contradictory, illogical point. If violence is a non-proportional response to political oppression, why is the US invading the oppressive state any less immoral than the people rising up against the state themselves? If it’s because it is “more effective” to have another state intervene, that doesn’t work. Whether a tactic ends up working or not is irrelevant in discussing whether it is a “just” tactic to undertake. Such a standard for “just” chops away intent and only leaves results. Intent matters. 

Furthermore, CON’s point ignores the simple fact that you can both revolt violently and request help from other nations, rendering this point non-unique. For example, consider the American Revolution where the Americans enlisted the help of both France and Spain. 

Even if that weren’t the case, we are analyzing whether violent revolution is a just response to political oppression in a vacuum. In other words, we are examining the relationship between a government and their people. Outside factors are not considered because they are unknown variables. 

PRO can demonstrate why pretty easily. For example, CON can not assume that an intervening state/organization exists. The western progressive interventionist ideology is a blip in history. For the vast majority of human civilization there would be no intervention. Where was the intervention during the Mongol massacres? Or the reign of Bloody Mary? Ivan the Terrible? Qin Shi Huang? Was resistance against those rulers immoral because of how the world turned out to be hundreds of years in the future? What about hundreds of years from now? This ideology will not last forever. 

Even if the judge didn’t buy that, western interventional power is severely limited. Kim Jong Un continues to hold power over North Korea and China continues to have concentration camps of Uigher muslims. Kim gets sanctions, but nothing power-toppling, and China gets nothing but a slap on the wrist and a new trade deal from the US and EU who are supposedly “bastions of liberty.”

Even the logistics don’t make sense. “The government will round up my family and kill them in an hour, but I can’t resist because the US may or may not come to my aid at some undisclosed time.”

And this is all ignoring the fact that simply relying on big daddy US or UN to intervene defies the idea of self-determination. Why shouldn’t the people be allowed to decide what they will permit, instead of it being decided for them by a foreign power?

A/2 III. Failures of Violent Revolution

First, RECALL & EXTEND: “Whether a tactic ends up working or not is irrelevant in discussing whether it is a “just” tactic to undertake. Such a standard for “just” chops away intent and only leaves results. Intent matters.” 

Second, the results of the study are highly questionable. In order to properly analyze which method works more, we would have to have two movements, one violent and one nonviolent,  for the same cause, of roughly the same size, and under the same regime to compare. Instead the authors simply took all movements throughout history and divided them into categories, ignoring the fact that the populace will elect to resist nonviolently before they attempt a violent resistance, and further ignoring that, as CON admits, there are more “slightly oppressive” regimes than there are “very oppressive” ones. In effect, as nonviolent means are elected more often and in far less oppressive regimes, their success rate inflates while the less chosen violent means in more oppressive regimes look bad in comparison. 

So while yes, in the Democratic USA, nonviolent protests can work wonders, if you tried a nonviolent movement in Nazi Germany as a Jew, you wouldn’t see much success. 

But that isn’t all. There are other, far more technical issues with the study. Pádraig Sinjun elaborates:

“They do not use revolutionary criteria for success, and in their mind the “Color Revolutions” and many other reformist, dead-end, or self-betraying movements were successful… They credit nonviolence with victory in cases where international peacekeeping forces, i.e. armies, had to be called in to protect peaceful protesters, as in East Timor, and they define victory simply as the achievement of a movement’s goals, as though movements ever had a consensus on their goals… They do not publish the list of campaigns and conflicts with their original study, and after extensive searching I was unable to find it. They explain that the list of major nonviolent campaigns was provided to them by “experts in nonviolent conflict”, in other words, people who are almost exclusively proponents of nonviolence... many of successful nonviolent campaigns on the list included armed and combative elements. The violent conflicts that they include in their study come from a completely different source: lists of armed conflicts with over 1,000 combatant deaths. In other words, wars. They are comparing apples and oranges, lining social movements up against wars, as though these different kinds of conflicts arose in the same circumstances and were merely a product of the choices of their participants. One methodological weakness they do admit to, in a footnote, is that by focusing on “major” nonviolent campaigns, they weed out the many ineffective nonviolent campaigns that never assumed large proportions.

He also notes: "They provide no evidence for key arguments like “the public is more likely to support a nonviolent campaign”

A/2 IV. Con...tradiction of revolution.

RECALL & EXTEND: Whether a tactic ends up working or not is irrelevant in discussing whether it is a “just” tactic to undertake. Such a standard for “just” chops away intent and only leaves results. Intent matters.” 

Weirdly enough, by arguing this point, CON affirms what PRO has said all along: violent revolution is a heavy, difficult matter. Now, this does not mean revolution can never be successful. 

There are clear examples of success
“Examples of successful rebellions are represented by the three major historical revolutions in France (1789), Russia (1917) and Iran (1979), as well as by the recent uprisings in Tunisia (2011). In all these cases, the pre-revolutionary government is overthrown and a new order is established.”

Howard Ryan states:
history provides abundant examples of violent uprisings winning mass sympathy, support, and participation. None of the armed revolutions and anti-colonial struggles of this century could have succeeded without the massive popular support they received. Observe the international support given in recent years to liberation struggles in Central America and Southern Africa. Some of the finest victories of the American labor movement have involved rallying public sympathy and support behind strikers who fought violently against police attacks. Examples were the three great city-wide strikes in 1934 in San Francisco, Minneapolis, and Toledo. Each won broad public support and each involved workers violently defending their picket lines--combined, of course, with nonviolent actions as well. If one's cause is clearly just, it isn't necessary to suffer peacefully the brutalities of the government in order to win popular support. To fight violently in defense of one's life, one's home, or one's freedom is a widely respected human right. In fact, where a movement faces heavy government repression, people may more likely join the struggle if resisters are fighting to defend their lives rather than allowing themselves to be beaten and punished. A campaign which emphasizes voluntary suffering may alienate rather than attract public support and participation.”

Note that a movement does not require extensive military hardware on its side, as guerilla tactics (which ironically are easier than the “traditional tactics” CON talks about to practice) can be extremely effective (for example, Vietnam against the US, American Revolution against the British, etc).

Howard Ryan finishes:
“The immense power of a popular movement should not be underestimated. As Malcolm X observed in 1964: The oppressor always points out to the oppressed, "The odds are against you." When Castro was up in the mountains of Cuba, they told him the odds were against him. Today he's sitting in Havana and all the power this country has can't remove him. They told the Algerians the same thing: "What do you have to fight with?" Today they have to bow down to Ben Bella. There probably has not been a successful mass revolt in this century where the ruling regime did not try to appear militarily invincible. The Vietnamese showed the "invincibility" of the U.S. military by defeating it.”

-------------------------------------------------------------------

Back to you, Undefeatable. 










Round 3
Con
#5
Contradiction of Violence 
1. Nonviolent oppression?
Unfortunately, Pro assumes that people have tried everything and will only use VR as a final resort. This is not backed by any sources. People are emotional, willing to lash out with violence supported only by natural assumptions. Pro's world assumes people are perfectly rational, but I have already proved that violence is so influential and dangerous that you forfeit your rights, just as the regime forfeits its ability to rule when it unjustly uses violence. Bringing back my BLM example: Certainly, the Civil Rights Movement has not brought upon efficient and incredible change. There is still structural racism in the US. However, it is difficult to say for certain that the BLM movement was just or even justified merely because M.L. King failed to convince change on the local level overall (despite Plessy v Ferguson and other supreme court cases displaying attempts to solve structural racism).

Now, I will note that my blanket statement that violence is never morally justified applies to an overall case (the vast majority of scenario). In the special case where it is used for self-defense, we must go to Refutation 5 (people are not moral agents under self-defense). And remark that this becomes an amoral problem. Here I will clarify with an expert's knowledge about my case which was laid out in layman terms. [Refer to Source 1]

Starting on page 308, the expert highlights how violence by its nature is harmful and can be noted by intention as well as effect. The unpredictable predictions link back to my idea that the slippery slope of revenge is difficult to understand. Cragg also points out that self-defense "implies that those who use violence to achieve their goals both ignore and undermine the moral status of those on whom their violence impinges" (p309). However, since the moral agent acts within the rules, the disrupting effect refutes the very idea of planning and understanding the consequence. This is further supported by Weede's note of revolutionaries failing to uphold the promised liberty, as well as the idea that you need good planning to succeed in a VR (coordination to bring down the regime).

Secondly, Cragg connects the violence back to the fact that the effects will threaten the safety of both parties. As such "violence gives an urgency to self-interest that if it comes to dominate removes the agent from the moral sphere" (p309). The violence was used in this case as a show of repression. As a result, it negates the moral status and becomes anti-moral rather than merely immoral. Both the regime and the person in question are using violence as means to an end. As such, Pro must differentiate the inherent rights of government enforcement, compared to the people's spoken power. Otherwise, he faces the problem of how both political oppression can be just, simultaneously while VR is just. If the oppression is unjust, then likewise the VR will be outside the realm of the moral sphere as the urgency forces a necessity that is beyond mere morality and now about evolutionary survival. 

In essence, Pro may justify supporting the VR (as ironic as that may be since the regime would surely disallow it). Cragg advocates for a system for non-VR of disputes whenever possible, and I see no inherent reason why a rational and clever regime would not simply go for this method of repressing violence. To win this, Pro must lay out why we have to accept the law's "just". After all, the definition of the just in this debate is based on morality itself. Man's rights and rights of laws must be inherently rejected by the regime so that we can talk about how self-defense is justified. Relating to the US which was the forefront runner of democracy to enforce equal rights has ironically fostered racism and caused such inequality that is impossible to directly contradict with normal-type laws.

Ultimately, even if Pro swings it back to a legal basis, he is forced to allow the people to determine the arbitrary ways the regime may attempt to oppress people. Indeed, while stronger regimes like Nazi put people in slavery camps, subtler regimes like China prevent freedom of speech and press, and the most subtle abuse of power is perhaps US cops kneeling on George Floyd. Pro can defeat the former case, but even the second not to mention the third becomes problematic when we accept that we must resort to violence when everything else seems too slow or too ineffective.

2. Some innocents are harmed

Pro notes some events in which war was conducted inappropriately but fails to link it back to an inherent cause. Is he cherry-picking, or is he noting that we conduct war with such ferocity that it is impossible to avoid innocent casualties? It is difficult to say for sure. By contrast, I note that VR's are harming their very family and friends (rather than potential allies) by coercing them through emotional manipulation and sometimes physical.

3. Violence prevents people from being sympathetic

Appeal to authority. Likely only applies to extremely obvious regimes, and otherwise encourages recklessly acting against subtler acts of oppression that may be more difficult to solve.

War vs Revolution

Pro points out some minor flaws with my argument while listing very old examples of revolutions that had no intervention while choosing to note the major examples where the US intervened (such as the Vietnam War, Chile, Latin America, Moro Rebellion, World War II, and Cold War). The modern world is very different now with the construction of the UN and the US accepting greater responsibility and attempting to make amends.

Pro's source also contradicts himself as it notes "The United States recently imposed sanctions on Chinese officials involved in persecuting the Uighurs and punished companies believed to be reliant on Uighur forced labor." 

The ICC is also trying to take action in a non-violent way: "Uighurs in the diaspora are pushing for the ICC to investigate China for genocide and other atrocities." There are also economic problems that prevented Trump to fully stop relations with China, nevertheless "Some US lawmakers have been pushing for the US to get tougher on China on the Uighur issue, and the State Department has advocated for the Uighurs as part of its religious freedom initiatives. And the Trump administration is finally beginning to take more forceful steps to punish China for its human rights abuses." They sanctioned multiple Chinese officials, displaying the fact that they were taking action. 

Recall that I do not have to prove that the wars work, only that either War or non-violent actions are more just than VR's (which, I remind voters, Pro repeatedly refuse to give examples of when they worked and why it isn't cherry-picking), reducing Pro's power over his side.

Failures of VR

While Pro makes a good case to attack non-VR's against my big source, he is left with nothing on his side. I will take this opportunity to stack on another source agreeing that non-VR's are better. A different expert named Howes has quite an amount of empirical evidence in favor. [2]

Firstly, the article establishes the difference between pacifism (passive obedience) compared to nonviolent civil resistance. From a philosophical standpoint, he notes how there are six methods to accomplish this: protest, social, economic -- boycotts and strikes, and finally intervention. The expert notes how the peace talk is not as effective, and that the failures must be differentiated from the civil rights movement which was not a pacifist. Though Howes also burrows from Chenoweth, he further talks of big events that were formed through crucial nonviolent events. The end of legally sanctioned slavery had to be achieved through non-violence, despite a war paving the path. The boycotting allowed abolitionists to push through and displays my point that even wars need the complement of non-VR to succeed. Next, the women's rights were also gained through the meetings, organizing, and managed to obtain their voting rights --without even one VR. 

Howes follows through with natural trends of the world leaning towards non-violence. He stacks on how "Joshua Goldstein argues that fewer wars are beginning, more are ending, and that the wars that remain are less lethal and smaller", as well as the fact that Steven Pinker talks about all violence in general. "Violence at every level—familial, tribal, neighborhood, and state—has declined the world over. Domestic violence, rape, murder... over not only 50 years but 500 years have been reduced." The UN peacekeeping has been crucial to reducing warfare and be able to prevent political oppression as well. 

Thirdly, Howes talks of the necessary traits for non-VR's to succeed-- the political leverage, and the ability to continue acting against the oppression. The gathering of people, and the contrast of dispersion of people, combined to flexibly resist against the regime. He concludes that "Schock's work is complemented by Wendy Pearlman's findings that there is an organic relationship between the degree of internal cohesion in the organizational structures of social movements and the likelihood and ability of movements to use nonviolence instead of violence." Recall that Pro has not negated on a logical and reasoning based on why non-VR's are better. The entry cost is lower, as the physical skills and activity requirements are less. The violent campaigns only increase the risk.

Recall my R1 argument talking about how the high level of commitment makes it difficult to truly be effective. Not only so, but Pro also fails to talk about how I said you can convince the opposing side more easily. The defection is much more willing, from Sharon Nepstad. [3] Pro speaks of how the Violence comparison is off but gives no constructive for VR's himself. By contrast, the narrative of the three big civil wars by Nepstad, combined with the analysis by Chenoweth still establishes non-VR's as relatively successful. Violence only unites the oppressors together, while non-VR encourages them to think otherwise. 

The next part completely dismantles the idea of violent oppression. It's largely known that the domestic public is necessary to support military power. Howes' review of Reiter and Stam 2002 notes, " We may discover that there is a tremendous disconnect between a state's access to technological, financial, and human resources and the social, political and other intangible factors that allow it to translate those resources into military power in war." From a fundamental perspective, it simply doesn't make sense for the government to randomly hit the people for no reason or keep them under slavery. After all, you need people on your side as well. If you only use power to oppress, you simply cannot have room for civil society. Even the regime would need informers and secret police to succeed. 

Finally, recall that Pro claims he advocates for VR in a vacuum, but his source directly contradicts him as the expert says you need a heterogeneous combination to succeed best. ("heterogeneous movements using conflictive methods and a diversity of tactics have been the most effective at seizing space and putting new social relations into practice.") So if you need both VR + non-VR to work, then VR alone cannot be just, even if we accept non-VR's are also unjust. 

Success of VR

Pro tries to assert that VR can succeed, giving a handful of examples. This is not relevant -- he must prove that they WILL succeed under the majority of circumstances. Otherwise, you are merely sending yourself off to your death, and people would not be willing to set up the VR in the first place. He has failed once again to refute my crucial problems:

  • Leadership and coordination (you can get public support, but can you get together before the regime finds out?)
  • Being branded as a terrorist (your people may accept, but will other governments support you after you take over?)
  • Access to weapons and Goods
  • Misinformation that makes it difficult to ascertain whether the oppression is truly severe enough
  • Battling trained soldiers while being untrained
Conclusion
My refutation is inherently linked to my constructive and I believe nothing has changed significantly.

1) There must be sufficient reason (fulfilled by Pro, BUT STILL problematic)
2) There must be a sufficient level of success (unfulfilled by Pro-- he only showed nonviolence relatively unsuccessful. If he would wave his burden of proof away then he refuses to look at consequences, and it would hardly matter if you managed to gain liberty or not.)
3) There must be no other option (point II's non-violent counterpart is not completely batted away from Pro's ideas)
4) The benefits must outweigh the negatives (unfulfilled by Pro)
5) The revolutionaries must be able to consider moral agents (challenged implicitly with 1's specific case of Self-defense, but unfulfilled by Pro)

At this point, I strongly recommend that voters look at how Pro fulfills his "reverse constructive" set upon my round 2, rather than how well he refutes my own constructive. Remember: VR may be superior to non-VR's while both are still unjust. "Lesser of two evils" does not negate the idea that something is evil (or in case of self-defense, inapplicable to morals). Pro's burden is harder to fulfill because he has to show VR against oppression is a moral based question, or deny it and set forth a new framework, and then show why it is moral at all.

Pro
#6
Thx, Undefeatable.

REFUTATIONS:

First, CON contests their BoP. PRO maintains that CON’s BoP is indeed to prove that violent revolution is unjust in all manner of cases. 

RECALL & EXTEND: 
an “on balance” standard would introduce absurdity into the equation. Under the standard, those who were being slaughtered would be forced to simply roll over because of the potential prospect of other people in other places abusing the privilege of being able to revolt violently.”

CON’s objection to this burden uses his own outcome-based standard, which PRO will later demonstrate to be flawed, making his objection moot. 

Next, CON lists requirements for PRO to fulfill, some warranted, some not.
These will be refuted in order.

A/2 #1: NEBULOUS OPPRESSION

CON's narrative is easily refuted here: if the oppression is subtle and nebulous, then violent revolution won’t even be in the playbook. 

RECALL & EXTEND:
There are an arsenal of options for people to take to combat political oppression, such as trying for reform and civil disobedience. Declaring war against the state is a heavy matter that puts the lives of the revolutionaries and their families on the line, people do not undertake such a risk without having exasperated all other options. Indeed, as soon as those peaceful methods inevitably fail in the most tyrannical of governments, or they appear to have no utility against them, violent revolution then becomes the last-resort in the arsenal of options of the oppressed.”

Regarding CON's BLM/non-oppressive gov argument, if the US government is not choosing to be racist towards black people, then it is not politically oppressive towards them. This renders CON’s criticism completely non-topical, as the resolution specifically excludes any government that is not objectively politically oppressive. This should be pretty obvious, as it is impossible to be revolting against a politically oppressive state in which there is no political oppression. All CON does here is prove that violent revolution is unjust in a just society, which is news to nobody.

He may be arguing that people could be convinced that there is more oppression in a government than there really is, but if the government were less oppressive than one worthy of violent revolt, surely it would reform given the threats of the populace, rendering this point moot. 

A/2 #2: SELF-DEFENSE & OUTCOME-BASED MORALITY

CON argues that unless you are dead either way, violent revolution is immoral to undertake (i.e. unless you are being physically slaughtered by the thousands, it shouldn’t be an option). There are so many issues with this, so bear with me. 

  • RECALL & EXTEND:
“While CON can pretend they don’t exist, the truth is, political oppression often takes physically violent forms. Under CON’s own self-defense framework, then, this instantaneously grants PRO the win."

  • While CON may retort to the last point by saying any action that escalates suffering in the short term is immoral, he yet again ignores that the aim of the short term escalation of violence is to protect the people from an unyielding structural violence that causes harm over many years. The long-term prospects inherently outweigh the short term harms.
  • Again, CON’s method of moral judgement relies on the examination of results instead of intent. RECALL & EXTEND: Whether a tactic ends up working or not is irrelevant in discussing whether it is a “just” tactic to undertake. Such a standard for “just” chops away intent and only leaves results. Intent matters.”  If we were to adopt this pseudo-morality, then Poland should not have resisted against the Nazis. Greece should not have fought valiantly against both Italy and Germany, and Ethiopia should not have resisted against a fascist Italy invading their homeland. He may respond with his self-defense exception here, but PRO has already demonstrated how structural violence is just as serious in the long-term as direct violence. This renders any such objection inconsistent logically. This has major implications. By extension, we can dismiss all of CON’s practicality based arguments. This rules his 2nd, 3rd, and 4th Contentions toothless.
  • Even further, by CON’s own arguments he does fulfill this standard either. Under his own admission, nonviolence is unsuccessful about 50% of the time (ignoring the data flaws in the study, obviously). And most of these examples are in reform movements in mostly unoppressive democracies, not movements to topple the ruling elite of truly oppressive nations (RECALL that, comparatively, nonviolence is used in less oppressive regimes compared to violence in more oppressive ones). Any nonviolent movement that seriously threatens the ruling powers is likely to evoke violent retaliation at the hands of the state, as it did in Communist China at Tiananmen Square and in El Salvador. In this regard, the negative is being hypocritical with this form of argument, not only is nonviolence far from being very effective, but it devolves into the very violence the negative has condemned. They may say, “we cause less so prefer us,” aside from being wrong, that’s not the point. The point is that their own standard of absolutist morality is so intrinsically flawed that even a nonviolent resistance is immoral by their own standard.
  • Even if the above point is completely rejected by the voter, and we used CON’s arbitrary standard, PRO would fulfill it quite easily. 
This is because, unlike what CON would have you believe, the resolution is not a dichotomy (i.e. it is not between “no violent revolution” and “nothing BUT violent revolution”). In fact, the resolution is simply:

PRO - People can use violent revolution
CON - People can’t

As you can see, there is absolutely no reason why, in a PRO world, both violent and nonviolent methods could be utilized within the same nation to overthrow an oppressive regime (similar to how, contrary to CON’s 2nd Contention, there is no reason a violent revolution can’t enlist the help of a foreign power to intervene. In fact, a violent revolution may actually incentivize intervention as it legitimizes the use of violence to help the movement as well as providing a clear faction to side with). Indeed, combining the two strategies happens all the time in successful revolution and reform movements.

Howard Ryan writes:
“Not only has violence often been used effectively in struggles that are mainly nonviolent, but nonviolent protests have also played an important role in armed revolutions. The armed struggle in Nicaragua would probably have never succeeded had it not included mass mobilizations through general strikes and countless local campaigns involving boycotts, nonpayment of taxes, student strikes, demands for constitutional rights, etc.”

At the very least, this renders CON’s 2nd, 3rd and 4th Contention’s non-unique as both PRO and CON can access those impacts. 
But further, the voter can actually turn those points in favor of PRO because PRO achieves those standards better than CON. A combination of violent and nonviolent resistance is inherently more effective than only nonviolent resistance.

As Howard Ryan states,

“Nonviolence works best where governments, for historic contextual reasons, cannot sustain prolonged repression. Conversely, where governments attack campaigns over long stretches of time, nonviolence encounters the greatest difficulty--campaigners become too beaten or demoralized to continue, or in some cases turn to violent self defense.”

As the voter can see, it is often the case that in cases of concerted, prolonged repression, nonviolent campaigns fizzle out over time. However, contrast this with the examples above. The combination of violent and nonviolent revolution was enough to topple even the most brutal regimes.

  • Even further, the voter can RECALL & EXTEND PRO’s R2 refutations of CON’s 2nd, 3rd, and 4th Contention. There is no good evidence that nonviolent revolution is more effective than violent revolution, therefore there is no reason for CON to be trying to leverage this standard against PRO. 
In fact, there is some research that concludes that elements of violence actually increases the chance of a movement’s success: “If unarmed violence, reactive violence and omitted cases are analyzed, nonviolent success rates are worse than formerly considered. Inclusion of 19th century (1800-1900) cases and previously unanalyzed cases from the 20th century reveals that nonviolent campaigns experienced success rates of 48%, whereas campaigns that adopted unarmed violence were 61% successful, and campaigns utilizing reactive unarmed violence were 60% successful, while 30% of fully violent campaigns were successful.”

Further note that this trend persists despite the effect PRO noted in R2: “nonviolent means are elected more often and in far less oppressive regimes, their success rate inflates while the less chosen violent means in more oppressive regimes look bad in comparison.”

A/2 #3: NO ALTERNATIVES

CON acknowledges that PRO meets this standard except for his argument of foreign interventions. RECALL & EXTEND the litany of responses PRO gave last round. An additional note is that this standard is better phrased as "the populace feels that they have no other option." 

A/2 #4: IF VICTORIOUS, THERE MUST BE A NET-POSITIVE OUTCOME

  • NOTE: CON’s source citing “Mr. Weede” should be thrown out until CON provides an accessible copy. The link provided does not work.
  • No revolutionary fights against an oppressive government with the intention of establishing a similarly or even more oppressive government. Once again PRO ignores intent and judges based on outcome. Thus, cross-apply all of PRO’s refutations of outcome-based arguments. Just as much as there is no guarantee that the revolutionaries will win, there is no guarantee that they will create a successful state either. That isn’t the point. The point is that, given a violation of the social contract between the people and their rulers, the people reserve the right to try and reform that government through violent means if they deem it necessary. 
  • RECALL & EXTEND: “Even further, by CON’s own arguments he does fulfill this standard either....Any nonviolent movement that seriously threatens the ruling powers is likely to evoke violent retaliation at the hands of the state, as it did in Communist China at Tiananmen Square and in El Salvador. In this regard, the negative is being hypocritical with this form of argument, not only is nonviolence far from being very effective, but it devolves into the very violence the negative has condemned.” 
  • The idea that violent revolution always devolves is highly contested. 
RECALL ResearchGate's finding that violent means were oftentimes more successful. Successful means “achieving their goals.” Knowing that no revolutionary fights for more oppression, it’s pretty clear violent revolution isn’t devolving into dictatorships left & right like CON claims.

But further, the idea that violence always devolves is based on a misreading of history. 

As Howard Ryan writes: 
“When violent revolutions fail to produce peace, democracy, and justice, nonviolent advocates frequently blame the violent methods of the struggle. They minimize the evidence of progress made by some revolutionary governments in alleviating the social violence of grinding poverty and brutal dictatorship. They also fail to take into account the circumstances in which most revolutions have taken place: poverty, illiteracy, economic despair, international threats and blockades, and raging internal conflicts. To ignore these factors while asserting that violence produced tyranny serves to limit our understanding of history and precludes our drawing lessons from past struggles for justice.”

RECALL that, comparatively, nonviolence is used in less oppressive regimes compared to violence in more oppressive ones

Expanding further on the idea of historical context, take CON’s Russia example.

Howard Ryan contests that the struggles of Russia post-revolution was less a result of violence, but more of historical context:

“The Russian revolution, which was built upon highly democratic workers' soviets (councils), immediately confronted a complex of problems including: 

• the devastating effects of World War I (Russia bore the heaviest costs of the war, including 2 1/2 million lives)
 • a prolonged and vicious civil war 
• capitalist sabotage of production and breakdown of the railways 
• an economic blockade from the West 
• a shortage of raw materials 
• the failure of expected worker risings in post-war Europe, which increased Russia's political and economic isolation.

... A balanced evaluation shows that armed revolutions, while not achieving democratic socialist ideals, have won immense gains for hundreds of millions of people. The Russian, Chinese, and Cuban revolutions went far beyond the societies they replaced in terms of meeting basic needs such as food, health care, literacy, full employment.”

A/2 #5: ARE PEOPLE MORAL AGENTS?

CON gives the flawed argument that since violence is inherent in nature/human beings (i.e. no different than your heart beating), violent revolution is not morally charged in either direction. He even tries to argue that people can not be moral agents because natural instinct and savagery will prevail above rationality. 

This argument is absurdly easy to refute. Note that all the human actions CON pegs as amoral are actions which are done unconsciously. Wikipedia defines a moral agent as "a being who is capable of acting with reference to right and wrong." To be capable of acting in reference to right and wrong entails the presence of a discerning consciousness and rationality. Indeed, that is what separates the moral action from the amoral one: consciousness. This is what shows the fundamental fallacy of CON’s argument. Making the decision to violently struggle against a regime (note that a “regime” is a highly abstract idea) requires a large amount of conscious ability. And that stays true of any choice to engage in violence. It had to be a conscious decision. Humans have instinct, yes, but that instinct does not override our ability to make choices about what is right and wrong.

This also highlights the difference between the man and the animal. For the man, the beating of the heart, breathing, and other acts that require no consciousness to accomplish are amoral. For the animal, all acts, even those of violence, are amoral acts done simply as a result of instinct. 

As for the rest of the metaphysical mambo-jumbo spewed in this section, it is either nonconsequential or de facto refuted.

Back to you, Undefeatable.













Round 4
Con
#7
Defense
it is impossible to be revolting against a politically oppressive state in which there is no political oppression
I disagree. See the American Revolution. There was very little oppression, yet we managed to justify “tax without representation” somehow as equivalent to oppression. See point 1 below for more evidence.

the resolution is simply:
 
PRO - People can use violent revolution
CON - People can’t

Voters should note that it’s “can and should”, while Con is simply “people shouldn’t, or cannot” because if something is morally correct then you ought to do it, not merely being capable of it.

 there is no reason a violent revolution can’t enlist the help of a foreign power to intervene
Pro misses out on the point when I say that the Nazis were too powerful and so we must resort to foreign power only, as VR’s become pointless sacrifice.

Pro now shifts his ideology to try to also include non-violent resistance. But this is not a policy debate where, for example, gun control may be supplemented by education and extra policies to reduce crime. Pro admitted previously that he is trying to analyze violence in a vacuum because many countries are unwilling to intervene on an international scale. So which is it? Are we considering the best case, where citizens need not waste their life, and the US and UN always will police the oppressive regimes? Are we considering the strictest case, where citizens are only using VR without other methods? Or is pro merely cherry-picking a specific case? The premise seems to note only VR. If there are situations where non-VR seems impossible (as Pro claims), why would we attempt the combination of both?

there is some research that concludes that elements of violence actually increases the chance of a movement’s success
Bear in mind that this research is very short and only points out the flaws of the study, including unarmed violence as potential effective, while ironically fully violent movements are only 30% effective. In addition, this does not concern the entire morality of VR's at all. The same researcher had a dissertation 5 years ago that looks far more in depth with the same topic. Within the conclusion, he admits that "there is a certain historical moral element in the practice of nonviolence...nonviolence was strategically interlinked with metaphysical notions ofhuman existence." (p298) Not only so, he even points out that the non-VR's transcend mere historical boundaries, defeating his current research on a morality level. Pro's expert points out that the massacre against non-VR's only propel it towards greater height, reducing the uselessness that Pro claims. Though there are many reasons necessary for success, Pro has not managed to prove that these are more difficult to fulfill than VR's. Indeed, Alexer highlights "nonviolent protests and social movements more generally, will be able to withstand severe staterepression and experience increased mobilization even after being repressed." (p301) Clearly, even if my research is flawed, when you reach the ontological basis, even Pro's source was forced to admit that non-VR's are more moral than VR's.

 CON’s source citing “Mr. Weede” should be thrown out until CON provides an accessible copy. The link provided does not work.

Pro decides to attack the source itself instead of my added logic to Mr. Weede’s ideals and my paraphrasing. He is already conceding my argument. If anyone wishes to verify my citation and arguments’ credibility, the access link is here.

Pro also says...
 in terms of meeting basic needs such as food, health care, literacy, full employment

But fails to think about economic growth, not to mention he conceded the grab of power from the revolutionaries. Indeed, there is greater equality overall, but it is difficult to say for sure this is an improvement. For one, you had to kill thousands of people, which means the pie to share the economy is lessened. Based on this logic, the more people killed the better. But obviously life cannot be exchanged for money. The "greater good" of the fewer people makes me feel that Pro is only advocating for even more bloodshed for the purpose of having more people have access to services. If there are 100,000 people left, that's more people to feed. If there's only 10,000 people left, they can receive even better benefits. With justification of self-defense on their side, I only worry about massive collateral damage, should the revolutionaries also be greedy and think about fulfilling the greater proportion of the people. In reality, it's possible for them to "cheat" by killing more people so that there's less people to take care of. For governments to succeed on an international level, they have to be economically well too. Replacing with a democratic institute seems much more respectable and reasonable to me here.

Pro drops my entire philosophical argument. He states:
that instinct does not override our ability to make choices about what is right and wrong.

As for the rest of the metaphysical mumbo-jumbo spewed in this section, it is either nonconsequential or de facto refuted.

He has completely failed to tackle the idea of urgency within the people. Notice how he completely drops the fact that the government also had its own ability to choose. When the people attack first, the government responds in kind with a justification similar to VR. But Pro would consider the government’s defense equal to more oppression stacked on top, rather than justified self-defense.

Pro has ignored my crucial quote that explains why self-defense is an amoral act:
 self-defense "implies that those who use violence to achieve their goals both ignore and undermine the moral status of those on whom their violence impinges" (p309).

Pro has ignored my slippery slope:
 A man takes a goat; the proprietor of the goat strikes the man's child; the man's child murders the proprietor's significant other, etc. until ... no one can recall how the viciousness even began.

Pro admits that democratic socialist ideals are not upheld. Hence, the revolutionaries did not truly desire or know-how to implement freedom. As a result, this proves my point that the urgency to defend yourself overcomes the ability to implement a free and better government once you win.

Reorganization
I will now crystallize and reorganize my argument in a chronological order to make things clear and why VR’s cannot be justified in the vast majority of circumstances. 

0. Philosophical basis: Pro notes how it is legally necessary for people to strike back, but this does not condone the moral justness of their actions. In this case, self-defense is an amoral aspect -- we can only resolve this on a legal basis. However, laws are non-applicable to VR’s, as the regime determines the rules, which is inherently challenged by the people. Therefore we must find some other basis. Pro has not provided the objective framework to determine the answer. He would then likely think of the people, the rights that we inherently have, regardless of regime or not. 
        Lack of true basis leads to Point 1 (see below), the idea that now people can unfairly overcome the government. Yet in more democratic societies we elect our leaders. So are they useless whenever they go against any view of liberty? If even half of the population disagrees with Trump’s immigration policy (as it unfairly oppresses the foreigners), can they violently revolt against the government?
         The logic of people having a say leads to a slippery slope where we can justify any violation of liberty as unfair. Pro would justify VR’s even in extenuating circumstances, such as the current virus and quarantine debate -- which, VR’s would be heavily counter-intuitive as the disease would only spread easier with people rallying and bloodletting. After all, non-VR’s are comparatively useless in this situation. The world we create inherently destroys the idea of government gaining consent from the people to control their actions. We advocate for allowing sacrifice for some rights for the gain of security. We determined our hired governors were more skilled, educated, etc. However whenever we doubt the situation has oppression beyond acceptance, we come up with the problem of the people also violating the social contract. Indeed, not only are they disagreeing, they are lashing back with harm. With pro’s ideals, we might as well have anarchy -- there are always going to be some people complaining they can’t do one thing or another. 
  1. Pre-Revolution: Pro has failed to refute my idea with people holding power they had previously agreed to give away. In more democratic societies the “oppression” is appropriately defined and always in an unfair way, especially since the people may decide ambiguously that inequality can be oppression. Or even something as vague as “taxation without representation” that the British had little idea the Americans valued. I already mentioned this was too vague to work, and I’ll detail precisely why here. *** Even our most famous revolution’s justification was dubious to think of.  *** Britain had begun a war and needed to pay for the costs while being in debt. It protected the American colonies and despite no “representation” the addition of taxes was entirely acceptable. The Americans enjoyed a great amount of liberty and were basically equivalent from being freed from tyranny. They claimed oppression while in reality only the negros on the plantation were in true slavery. Historians widely “criticize American Whigs for their highly inflated prattle about the "slavery" resulting from British policies” [1]. The colonists never wanted to be represented (as they would simply be outvoted), and would never agree to being taxed (as it is more practical and convenient otherwise). Therefore, Pro’s argument is clearly wrong that the oppression is always obvious, agreed on a universal level, and the VR response is “just”. The people’s judgement were biased and this point stands. The colonists claimed “tax without representation” yet failed to reaffirm the right in the constitution. It seems to me that the people will use any excuse to justify the VR, oppressed or not. Even if we say this is cherry-picking, more morally ambiguous charges can still be used to justify VR’s, making the self-defense argument completely moot.
  2. Planning Revolution: Pro has given examples of ways that people have managed to succeed but has not defeated any of my reasoning why these may be outliers. It seems illogical to me that the nonviolent resistance with far more participants would fail, while VR with the only fit, skilled and daring fighters would somehow succeed. His source tells us that we must combine both for the idealized resistance. He has not explained how this connects to VR’s in a vacuum.
  3. During Revolution: Pro has dropped the impacts on their people. With the pre-revolution emotional manipulation in place, the revolutionaries are also likely to harm other innocents and attack the wrong people. Consider that, the military is only following orders and that my argument about defection powerfully sways people to your side. It takes severe pressure and motivation to attack non-violent people.
  4. Post-Revolution: Pro did not refute the idea that the rational planning necessary for the new government to succeed is inherently gone in the mind of the violence that was so urgent to win. As the revolutions only lead to the same cycle recurring, Pro advocates for a world of endless slaughter and no real solution. This is the crucial difference between self-defense and VR. Hell, Pro advocates for the people’s own power without depending on others for help, yet his own source in R2 that lists the four successes finds “without external intervention, the initial configuration of precision parameters would have led to a rebellion followed by a return to the status quo”.
Each of these points contributes to the overall unjustifiable nature of VR. Combined together, they destroy Pro’s argument. 

When oppression is too subtle, you risk the problem of too much liberty, power in the people, and fighting for the wrong reason. What has BLM truly managed? Was it more significant than the civil rights movement? Was it worth the sacrifice of lives? Pro has failed to answer these questions.

When oppression is too much, you risk dying as a martyr and it is incredibly difficult to set up the revolution in the first place. The urgency to take down the Nazi’s lead to this being an amoral necessity rather than a just action.

Finally, when oppression is just the right amount, you still can’t resolve the inherent problems of the irrational people. The nature of violence in the hands of the fighters encourages you to become corrupted and to take power. There is no obligation to support the people who have not joined you in the VR. Real-life examples I’ve highlighted display this problem. You are encouraged to repeat what the dictator did -- you know the power of violence. And so you will recur the oppression of the people until no one is left, or Gandhi steps up and stops this endless cycle.

What kind of world is left if we accept Pro? Revolutionaries could say the cops in America are just as bad as Hitler. They target black men with an unacceptable level of oppression. The civil rights movement has been too slow; therefore we must use violence. We revolt together and are pushed down both by the government and other foreign forces. Say a miracle occurs and we succeed despite branded terrorists. The new government is likely an oligarchy with a much poorer balance of power than the three branches. Why keep the old method if the red tape got in the way of slow legislation? Nevertheless, we would have little idea what laws to implement to treat racism. How can anyone both be able to commit violence and know politics? The two don’t go together. We realize that people are just as racist as before. We solved nothing, and countless people died. 

In the end, is VR a just response? No; it is just a response
Thanks to MisterChris for the debate.

[1]  full article is in pastebin


Pro
#8
Thx, Undefeatable.

REFUTATIONS OF R3:

A/2 “1. Nonviolent oppression?”
 
  • “People will abuse violence”
It is true that violence is a symptom of emotional anger: anger at injustice. But as much as there is anger at an unjust regime, there is fear and desperation. Fear of what sort of suffering war with the regime would entail, and desperation as people suffer under a brutal system that can often not be effectively be fought through nonviolent means.  People are not rabid beasts, craving violence and suffering as CON would portray… In order to spark violent revolution, the anger and desperation of the people must outweigh the fear of what a violent revolution would entail. People aren’t stupid. They know that if they raise weapons against their government, they will suffer the penalties of war.

And let’s not confuse a violent revolution with a violent movement. BLM is not a revolutionary group that intends to supplant the US government, they are a movement that aims to reform the US government. Waging war against the state and rioting for a cause are two entirely different things with two very different sets of consequences. To use BLM as an example to say violent revolution will be abused is faulty at a fundamental level. 

  • “Most revolts are not justified” (Craigg)
Craigg argues that violence must be judged in a vacuum, and that it is wrong in a vacuum because it infringes the human right of life. 

Yet, Craigg’s argument ignores that context outside a vacuum matters. If we’ve established a relationship of oppression by one party, as we have in this resolution, things are more complicated. If human life has intrinsic value like he posits, there is therefore an intrinsic moral responsibility to protect it and prevent it from being devalued by oppressive forces in the long run. 

RECALL & EXTEND that if even a few scenarios justify violent revolution, PRO’s BoP is instantly fulfilled.
RECALL & EXTEND that the relationship of oppression entails violence, whether physical or structural.
RECALL & EXTEND that, by definition, even the revolutions against non-physically violent yet oppressive regimes are self-defensive in nature. 
RECALL & EXTEND that the Responsibility-Sensitive Account of Proportionality as posited by Zsolt Kapelner states that a populace that is being repeatedly targeted with structural violence would be justified in a more substantial response to cease the otherwise unobstructed pattern of indirect violence. 

Indeed, as Howard Ryan argues, 
“The notion that violence is inherently evil is problematic because it assumes that a violent act may be judged apart from its circumstances, intentions, or consequences. By the moral logic of many pacifists, it would be an evil act to kill one person to prevent that person from killing ten or a thousand persons”

  • “Cases of self-defense are amoral”
RECALL & EXTEND that any decision made by a conscious mind has moral ramifications. By definition, self-defense must be on the moral spectrum. To win this point, CON must be able to prove that humans do not make the conscious decision to engage in violence. 

A/2 “2. Some innocents are harmed”

CON gives virtual non-response. PRO’s refutation stands unrefuted: 
“The problem with his argument is that it is a moral absolutist view, in which any collateral damage by the means invalidates the ends, even if the ends outweigh the means in the long run. War is not fun for anyone involved, and there will be many individual injustices in any war. By this logic, no war is justified. Instead, we should adopt the view of situational morality, where the attempted ends justify the means.”

A/2 “3. Violence prevents people from being sympathetic”

Another virtual non-response other than the statement that PRO’s point only applies to regimes in which violence is obvious. This point is pretty moot, as we’re discussing regimes in which their transgressions are severe enough to warrant overthrow in the eyes of the populace. If it’s obvious violence would be unproportional to the transgressions, there won’t be a push for regime overthrow to begin with. 

A/2 “War vs. Revolution”

CON fails to grasp PRO’s refutations, and essentially continues arguing with the same premise that PRO has already debunked.

RECALL & EXTEND all of PRO’s R2 refutations for this point.

A/2 “Failures of VR”

  • First and foremost, note that CON continues to use his flawed outcome-based moral model. 
RECALL & EXTEND: Whether a tactic ends up working or not is irrelevant in discussing whether it is a “just” tactic to undertake. Such a standard for “just” chops away intent and only leaves results. Intent matters.”

RECALL & EXTEND: “by CON’s own arguments he does fulfill this standard either...not only is nonviolence far from being very effective, but it devolves into the very violence the negative has condemned. They may say, “we cause less so prefer us,” aside from being wrong, that’s not the point. The point is that their own standard of absolutist morality is so intrinsically flawed that even a nonviolent resistance is immoral by their own standard.”

  • Second, RECALL & EXTEND: “Even if... we used CON’s arbitrary standard, PRO would fulfill it quite easily.”
RECALL & EXTEND PRO’s R3 point that the resolution is not a dichotomy. 
“there is absolutely no reason why, in a PRO world, both violent and nonviolent methods could be utilized within the same nation to overthrow an oppressive regime. Indeed, combining the two strategies happens all the time in successful revolution and reform movements.”

This effectively undercuts the CON narrative that the success of nonviolence is unique to his case. Now, there is the objection of “violent revolution can not contain nonviolent resistance by definition.” However, that is ridiculous. A violent revolution is a movement where violence is used to overthrow the state. Where in that definition is nonviolence excluded? Indeed, “violent revolution” is the inclusion of violent tactics, not the exclusion of nonviolent ones.

Peter Gelderloos writes:
“I know of no activist, revolutionary, or theorist relevant to the movement today who advocates only the use of violent tactics and opposes any usage of tactics that could not be called violent. We are advocates of a diversity of tactics, meaning effective combinations drawn from a full range of tactics that might lead to liberation”

RECALL & EXTEND that “there is some research that concludes that elements of violence actually increases the chance of a movement’s success...
 Further note that this trend persists despite the effect PRO noted in R2: “nonviolent means are elected more often and in far less oppressive regimes, their success rate inflates while the less chosen violent means in more oppressive regimes look bad in comparison.”

  • As for defectors, Howard Ryan addresses this point extensively:
“It is more often the case that police and troops, when ordered to do so, beat nonviolent protesters, and they sometimes beat them with cruel vigor. Even where troops have refused orders to attack nonviolent protests, these tend to be exceptional moments within a larger context of repression…

While disobedience of government troops during nonviolent campaigns is atypical, mutinies and defections are actually commonplace during armed revolutions. The struggles in Russia, China, Vietnam, Cuba, and Nicaragua were all aided by demoralization and defection of government troops. The Russian revolution was notable for being nearly bloodless, as the Bolsheviks enjoyed wide sympathy among the rank and file of the army and navy, who offered little resistance to their takeover in October 1917.”

  • Addressing CON’s reasoning for why nonviolence works better, he continually ignores that there is a limit to what nonviolence can accomplish. While it’s true that nonviolence is easier to execute, it comes with some drawbacks that make it impractical in the face of true oppression.
Howard Ryan writes:

“Nonviolence works best where governments, for historic contextual reasons, cannot sustain prolonged repression. Conversely, where governments attack campaigns over long stretches of time, nonviolence encounters the greatest difficulty--campaigners become too beaten or demoralized to continue, or in some cases turn to violent self defense. In leading India's movement against British rule, Gandhi was regularly disappointed because the masses were unable to maintain "nonviolent discipline" in the face of government brutality. The few thousand specially trained satyagrahis tended to take their blows without complaint. But the hundreds of thousands of peasants and workers who rose up in struggle did not favor peaceful suffering and preferred to defend themselves when under attack. Gandhi and his colleagues did not offer the Indian people an organized strategy for defending themselves. Hence, the colonial government was, by and large, able to contain Gandhi's movements through a combination of arrests and beatings.”

Finally, while CON can list the problems facing the initiation of a violent revolution, he can not argue that there have been countless examples of people who did overcome those problems out of necessity.

CRYSTALLIZATION & SUMMARY:

This debate comes down to a few key points.

A. OUTCOME-BASED MORALITY

This was one of the biggest underlying assumptions of the CON side throughout the debate: that the outcome is all that matters in deciding whether an action is moral. That we should prevent the oppressed from taking violent action to embetter their position because they are likely to fail (which is, ironically, a symptom of their oppression).

Judge, if you buy this pseudo-morality, you will buy into a CON world where even the most noble of intentions can not justify acting when failure with consequences is a possibility. The Jews would be told to sit idly as their friends and family are killed by a brutal regime in front of them. The Polish would be made to allow Nazi Germany to take over their land and enslave their populations without resistance. Greece would be condemned for resisting as both Italy and Germany invaded their lands, as would Ethiopia as Italy aggressively colonized their territory. Indeed, so CON argues, why should these people fight against tyranny? Why would they make their situation worse by resisting when they are likely to lose? 

It’s obvious how appalling and dangerous this line of thought is (well, dangerous to everyone except the oppressors. Indeed, if they are militant enough, no one can morally resist them!). The morality of violence is not about success. It’s about the oppressed refusing to passively allow their lives to be systematically devalued by their oppressors. If they believe that cause is worthy of waging war, who are we to tell them otherwise?

Indeed, PRO has repeatedly challenged this line of thinking throughout the debate, and yet CON is still silent in defending it. This makes the majority of the debate absurdly easy to judge: with CON’s outcome-based morality refuted, CON loses more than half of their case (Contentions 2-4) in one fell swoop. We are then left with CON’s first contention as the only point left for PRO to contend with. 

Even if the judge buys none of this, he can STILL drop CON’s Contentions 2-4, because CON does not manage to prove that nonviolence is mutually exclusive with violence, nor does he prove that nonviolence is even more effective at all. 

B. PROPORTIONALITY

Perhaps the most important point of this debate is the question of whether violent revolution is a proportional response to oppressive governments.

CON would argue that the answer is a definitive “no,” as of course violence is inherently bad, right? 

Yet, judge, if you buy this form of moral absolutism, then as Howard Ryan writes: “it would be an evil act to kill one person to prevent that person from killing ten or a thousand persons” 

Indeed, if we’ve established that human life has innate value, then people should have the right to protect themselves or protect others from their lives being destroyed or devalued. In response, CON admits that self-defense from direct violence is justified, but of course since most revolutions are not done in response to direct violence, they can not be moral. 

Aside from the fact that it is untrue that most revolutions are not done in response to direct violence, here CON has already conceded the win to PRO. If there is a time where violent revolution is justified, then CON can not justifiably prevent violent revolution from being an option for the oppressed. Under such an “on balance” standard, CON would tell the Jew under Nazi Germany that they must suck it up, because if we allowed them to revolt violently too many would misuse such a privilege. 

Already, this debate is sealed in PRO’s favor. But even if you buy none of that, judge, remember that the entire premise that self-defense is limited to direct violence is flawed.

Indeed, violence includes structural violence, which is “the increased rates of death and disability suffered by those who occupy the bottom rungs of society, as contrasted with the relatively lower death rates experienced by those who are above them.” 

As the Responsibility-Sensitive Account of Proportionality posited by Zsolt Kapelner states: a populace that is being repeatedly targeted with structural violence would be justified in a more substantial response to cease the otherwise unobstructed pattern of indirect violence. 

Stanford furthers:
“Why does structural violence not merit the same urgency (as physical violence)?
If anything, the urgency is only greater. As opposed to a single punch or gunshot, structural violence is unwavering...You can disagree with the strategic efficacy of their actions, but when people defy — even violently — this endless structural violence, the only consistent position is to label their defiance self-defense.”

With CON’s admission that self-defense is justified, then, PRO wins. 

Even CON knows this, as he comes with a final hail-mary attempt at garnering the win: arguing that self-defense is neither moral nor immoral. That it is simply amoral.

Yet, this is such an easy argument to refute. RECALL & EXTEND that “all the human actions CON pegs as amoral are actions which are done unconsciously. Wikipedia defines a moral agent as "a being who is capable of acting with reference to right and wrong." To be capable of acting in reference to right and wrong entails the presence of a discerning consciousness and rationality. Indeed, that is what separates the moral action from the amoral one: consciousness.”

As long as the decision to resist is done by the conscious person, it can not be an amoral decision.

This leaves your decision pretty straightforward, judge: there is literally no way CON won this debate. 

Vote PRO.